5
$\begingroup$

A result of Selberg (A. Selberg. On the normal density of primes in small intervals, and the difference between consecutive primes. Arch. Math. Naturvid., 47(6):87–105, 1943) says essentially

$$\int _1^x(\psi (t+h)-\psi (t)-h)^2dt\ll xh\hspace {15mm}\psi (x):=\sum _{n<x}\Lambda (n)$$

on RH - see for example Lemma 1 of https://hal.science/hal-00289430v2/file/Goldbach4.2.10.pdf.

I have tried to find a proof of this result but been unable to locate one, and can't seem to find/access Selberg's paper. Does anyone know where I could have an idea of the proof? Thank you in advance!

$\endgroup$

1 Answer 1

10
$\begingroup$

The entire discussion below, as well as Selberg's result, is under the assumption of the Riemann Hypothesis.


A proof is sketched in Soundararajan's expository notes "The distribution of prime numbers", see pages 9-10 in the arXiv version. The explanation there is lucid and there is nothing I can add.

Selberg's 1943 proof can be found in his collected papers, specifically it is paper no. 12 in his "Collected Works. Volume I" from 1989.

In page 10 of his paper (page 169 of the volume), you'll see that he essentially proves the following: $$\int_{1}^{\infty} y^{-20/\log T}\left|\psi\left(y+\frac{y}{T}\right) -\psi(y)-\frac{y}{T}\right|^2 \frac{dy}{y^2}\ll \frac{\log^2 T}{T}$$ for all large enough $T$. If in Selberg's result you only integrate from $x$ to $2x$, and rename $y$ to $t$, you obtain the following direct consequence: $$(\star)\,\int_{x}^{2x} \left|\psi\left(t + \frac{t}{T}\right) - \psi(t) - \frac{t}{T}\right|^2 \ll x \frac{x}{T} \log^2 x$$ holds as long as $\log x \asymp \log T$.

In the result you are after, $h$ plays the role of $t/T$ as well as of $x/T$. However, $t/T$ varies as $t$ varies, while $h$ does not, so setting $T:=t/h$ does not recover the result you desire.

Fortunately, Saffari and Vaughan, in Lemma 6 of "On the fractional parts of $\{x/n\}$ and related sequences. II" (1977), found a simple way to recover the result that you state as a direct consequence of Selberg's bound. Namely, Lemma 6 proves the bound you are after, using the following elementary general bound implicit in the proof of their lemma (see page 25): $$\int_{X}^{2X} |F(x+H)-F(x)|^2 dx \ll \sup_{\theta \in \left[ \frac{H}{3X},\frac{3X}{H}\right]} \int_{X}^{3X} |F(u+\theta u)-F(u)|^2 du$$ for any $1\le H\le X$ and any square-integrable $F$. This immediately implies the result you ask about given Selberg's result.

Two remarks:

  • At least as I wrote it, combining Selberg with Saffari-Vaughan recovers the result you desire only in the range $h\le x^{1-\varepsilon}$ for some $\varepsilon>0$. However, your result stays true for larger $h$, as observed by Saffari-Vaughan in their Lemma 5 (see the lemma and the comparison with Selberg immediately after its statement). The general and correct form of the result you seek, as stated in Lemma 6 of Saffari-Vaughan, is $$(\star\star)\,\int_{1}^{x} \left|\psi(t+ h)-\psi(t)-h\right|^2 dt \ll x \left(1+\log^2(x/h)\right)$$ for $0< h \le x$, which they deduce from the following estimate (proved in Lemma 5): For $T\ge 1$ and $x\ge 4$, one has $$\int_{x}^{2x} \left|\psi\left(t + \frac{t}{T}\right) - \psi(t) - \frac{t}{T}\right|^2 \ll x \frac{x}{T} \log^2 (2T)$$ without the restriction $\log T \asymp \log x$ I imposed when deducing $(\star)$ above.
  • Shaving a log from the right-hand side of $(\star\star)$, even for some range of $h$, is in fact a big open problem, which is expected to be true in view of Montgomery's famous Pair Correlation Conjecture, which in particular implies that $$\int_{1}^{x} \left|\psi(t+ h)-\psi(t)-h\right|^2 dt \sim x \log(x/h)$$ holds for $1\le h\le x^{1-\varepsilon}$ and $x\to \infty$. For more on this see Goldston and Montgomery's "Pair correlation of zeros and primes in short intervals" (1987). Section 9 of Goldston's lecture notes "Notes on Pair Correlation of Zeros and Prime Numbers" is also relevant.
$\endgroup$
3
  • $\begingroup$ Do you know how Sound argues away the zeros with $|\rho |\geq x/h$? $\endgroup$ Commented Nov 27 at 17:18
  • $\begingroup$ @tomos Take a look at Theorem 28.6 in Montgomery and Vaughan's volume III ( personal.science.psu.edu/rcv4/Vol3/Vol3.pdf ) for a full treatment of the unconditional version of Selberg's bound; it can be adapted to give the RH-conditional bound as hinted by Exercises 1 and 2 in the exercise list after the theorem. $\endgroup$ Commented Nov 28 at 9:40
  • 1
    $\begingroup$ Wow, I had no idea there's a Volume III - today will be fun:) Thanks for these references! (My first comment saying so seems to have disappeared). I think that completely gives me my answer. $\endgroup$ Commented Nov 28 at 10:14

You must log in to answer this question.

Start asking to get answers

Find the answer to your question by asking.

Ask question

Explore related questions

See similar questions with these tags.